Whats wrong with D
Why is D not the best answer?
Anthony-Resendes on March 23, 2023
  • June 2007 LSAT
  • SEC3
  • Q13
2
Replies
Question about diagramming a premise
"They have no moral rights or responsibilities." In regard to the previous sentence, should this ...
rinavaleriano on March 23, 2023
  • June 2007 LSAT
  • SEC3
  • Q16
1
Reply
The Necessary?
I fail to understand what the "necessary" part is and by extension how that is reflected in the a...
Jerome on March 23, 2023
  • June 2007 LSAT
  • SEC3
  • Q17
2
Replies
Explanation for eliminating E, please
Are there more than two ways - literal and metaphorical - to interpret things? If literally ...
Mazen on September 19, 2022
  • June 2007 LSAT
  • SEC3
  • Q16
2
Replies
Explanation
Could someone please explain the answer choices and why B is correct?
Gretchen on September 19, 2022
  • June 2007 LSAT
  • SEC3
  • Q18
1
Reply
Error
I was reviewing my answers and there are two points were my answer was changed from correct to a ...
juma on March 18, 2022
  • June 2007 LSAT
  • SEC3
  • Q9
2
Replies
help
I picked D, I felt that was a flaw in the argument
chriscole on February 5, 2022
  • June 2007 LSAT
  • SEC3
  • Q25
8
Replies
Why is answer choice B wrong?
I cannot really differentiate between B and D (d being the correct choice)
Nishant-Varma on June 3, 2021
  • June 2007 LSAT
  • SEC3
  • Q23
3
Replies
Can we diagram this question?
Hello can we diagram this question to: ~comfort --> sake of appearance contra: ~sake of a...
cheynnelee on January 4, 2021
  • June 2007 LSAT
  • SEC3
  • Q1
1
Reply
Is this questions diagrammable?
Is this question diagrammable to: sheep farming v ~hard evidence --> Extinct contra: ~...
cheynnelee on January 4, 2021
  • June 2007 LSAT
  • SEC3
  • Q9
2
Replies
Ans Choice B
Hi, could you please explain why B is wrong Thanks.
ankita96 on September 5, 2020
  • June 2007 LSAT
  • SEC3
  • Q10
2
Replies
Q7
I'm not clear with this question. Can you please help me understanding this Question in a better ...
Saksham-Sabarwal on September 3, 2020
  • June 2007 LSAT
  • SEC3
  • Q7
2
Replies
Answer choice B
How come B cannot weaken the argument? Pleas explain
delacruzcindy4 on August 14, 2020
  • June 2007 LSAT
  • SEC3
  • Q15
2
Replies
How is the answer D, and not E? They both seem ...
Jessw on August 13, 2020
  • June 2007 LSAT
  • SEC3
  • Q12
6
Replies
Request for Differentiation
What is the difference between "C" and "D"?
talk2sarahw on August 12, 2020
  • June 2007 LSAT
  • SEC3
  • Q5
6
Replies
Can you negate answer choice B
In order to get the right answer for necessary assumptions, I use the denial test. I use it for a...
RS1 on July 25, 2020
  • June 2007 LSAT
  • SEC3
  • Q17
2
Replies
Flaw
What flaw is this one?
LeeLarue on July 19, 2020
  • June 2007 LSAT
  • SEC3
  • Q23
2
Replies
Missing premises drill
I’m having trouble finding the first and second premises. I seem to get it right when it’s case o...
joniqueac on June 28, 2020
  • June 2007 LSAT
  • SEC3
  • Q2
2
Replies
Answer A
Watched the explanation. Still confused as to why "A" doesn't work. Help please :-)
AMS on March 13, 2020
  • June 2007 LSAT
  • SEC3
  • Q9
2
Replies
Why C actually weaken the argument
I get why the answer is not B, but I saw the video and really don't get the correlation between C...
Maria-Marin on February 8, 2020
  • June 2007 LSAT
  • SEC3
  • Q15
1
Reply